LSAT and Law School Admissions Forum

Get expert LSAT preparation and law school admissions advice from PowerScore Test Preparation.

 Administrator
PowerScore Staff
  • PowerScore Staff
  • Posts: 8917
  • Joined: Feb 02, 2011
|
#22829
Complete Question Explanation

Weaken. The correct answer choice is (D)

Because soft drinks and pay phone calls cost the same amount in 1970 and the price of soft drinks has doubled since then, the editorial concludes that phone companies should be allowed to increase the price of pay phone calls as well. This argument is quite weak, as the price of each product or service is typically determined by a number of factors, including cost of production, demand, and so on. The author offers no reason for assuming that the variables affecting soft drink prices are positively correlated with the variables determining the price of pay phone calls. If millions still drink Pepsi but demand for pay phones has decreased as a result of cell phone use, it is quite reasonable to suspect that one will cost more than the other. Likewise, if the cost of transmitting a phone call has decreased at a greater rate than the cost of manufacturing a can of Pepsi, using a pay phone today will likely cost a lot less than drinking a can of soda.

Answer choice (A): We have no reason to believe that the price of the product supplied by a machine is affected by the price of the machine itself. Even if it were, this answer choice only compares the cost of pay phones to the cost of soft-drink machines in the 1970's. It is unclear how such comparison would explain why the price of pay phone calls should be higher today. This answer choice is incorrect.

Answer choice (B): This answer choice does the exact opposite of what is needed. If inflation has caused the prices of most goods, including telephone equipment, to double since the 1970s, it would be reasonable to allow phone companies to adjust for inflation by increasing the price of pay phone calls. This answer choice is incorrect.

Answer choice (C): If government regulations of phone call prices are no more stringent today than they were in the past, phone companies have one less obstacle to overcome if they wish to raise prices. However, this answer choice provides no reason as to why pay phone prices should increase. This answer choice is incorrect.

Answer choice (D): This is the correct answer choice. If the cost of producing a soft-drink has increased at a greater rate than the cost of making a phone call, it is reasonable to expect that a soft drink from a vending machine purchased today will cost more than making a call from a pay phone.

Answer choice (E): At first glance, this may seem like an attractive answer choice, as it implies that any increase in the price of pay phone calls is justified by the higher level of technological sophistication behind telephone equipment. However, higher levels of sophistication do not necessarily equate with increased production costs: it is entirely possible that the technological advances described here have made the cost of telephone equipment lower than it was in the 1970's, thus pushing down the price of pay phone calls. This answer choice is incorrect.
 Sdaoud17
  • Posts: 85
  • Joined: Apr 13, 2013
|
#9211
Okay what the heck with this question lol? It made me :-? every time I do it and I dont get the right answer so here is how I approached it :

So in the stimulus says that since 1970 the phones were 25c each and soft drink from the vending machines were ABOUT the same (which brought a question in my mind , is "the same = about the same ") , I thought not .

then the Author made a silly conclusion which had NO correlation whats so ever that because soft drinks increased in 1990 to 50c (doubled) then the Phones should be allowed to raised.

Now when I read this and I made that confused face :0 of where is the connections between the both, I knew where to hit because it was clearly i had many reasons to hurt the conclusion. I moved to the answers

Now in my first time looking at the answers I was thinking the following :
A: Phone $ < Drink $ in 1970 (it doesnot help me much because as I said before I thought "the same" is NOt Equal to "about the same" which means the Answer is repeating the first premise again/ I thought it strength which wrong )

B: When I read it I was like A strength , No matter if inflation

C: I liked it but didnot love it !

D: When I read it I was like Okay Its like A and B which made cross it and move to

E: it doesnot affect it .

So I choose C which was Wrong . so in my second time going over the stimulus and see what else am I missing here , I starting having second thoughts now or you could say that I looked at the Stimulus as C-F relationship :
C: Drinks went up double ------> F: Phones Should too

I started thinking that I need to find another reason / C
or another F
or find away to switch the C-f --> f-c

so when I read the Answers again I picked
A, because I thought the wording "the same " is not really equal to "about the same"

Which was wrong

then in my third trial :
I was between B and E and here is my reasons :
I picked B Because I had another Cause or anther reason of that shows that because of the inflation Phones should go high which weakens or link the silly correlation that the author had

E: its another Cause or reason of why the phones should increase .

Both answers looked right to me .

after all this long story I would be appreciate if you help me out here , the online Explanation did not show how I read the stimulus was wrong .

Thank you :D
 Nikki Siclunov
PowerScore Staff
  • PowerScore Staff
  • Posts: 1362
  • Joined: Aug 02, 2011
|
#9214
Hey man,

I think you misread a portion of the stimulus, which explains why you also misunderstood many of the answer choices. The argument can be broken down as follows:

Premise 1: In the 1970's, local pay phone calls cost about the same as soft drinks from vending machines did.

Premise 2: The price of soft drinks has doubled since then.

Conclusion: The price of the local pay phone calls should be raised as well.

The argument proceeds by virtue of analogy: the cost of A doubled, so the cost of B that used to cost the same as A should double as well. Such arguments are not inherently flawed per se, but it all depends on whether the comparison between A and B is reasonable. In this case, you're correct: there is no reason why the prices of local phone calls and soft drinks should increase at the same rate. The correct answer choice to a Weaken question should highlight a difference between the two that makes the analogy fall apart. This, in turn, would suggest that phone call prices should not increase at the rate the author suggests.
A: Phone $ < Drink $ in 1970 (it doesnot help me much because as I said before I thought "the same" is NOt Equal to "about the same" which means the Answer is repeating the first premise again/ I thought it strength which wrong )
  • Read carefully! The stimulus is about the cost of pay phone calls and soft drinks, not pay phone machines and vending machine. The comparative cost of the machines is entirely irrelevant to the price of the products they allow us to consume
B: When I read it I was like A strength , No matter if inflation
  • Wrong, inflation does matter. If the price of most goods more than doubled, it becomes somewhat reasonable to expect that the cost of pay phone calls should double as well. This answer choice actually strengthens the argument.
C: I liked it but didnot love it !
  • Again, if you liked this answer choice that means you're going for the Opposite answer. If regulation of phone call prices did not become more stringent, we can expect that these prices would be determined primarily by the free market. Assuming that the price of other goods increased during that time, it is reasonable to expect that the phone call prices would increase as well. Like (B), this answer choice strengthens the argument and is incorrect.
D: When I read it I was like Okay Its like A and B which made cross it and move to
  • (D) is nothing like A and B. If the cost of ingredients for soft drinks increased at a greater rate than the cost of telephone equipment, it is reasonable to expect that soft drink prices would also increase at a greater rate than phone call prices. This is because the cost of these goods is, at least in part, determined by the cost of the ingredients or equipment. If true, (D) shows a crucial difference between the two commodities that form the basis for the author's analogy, and therefore weakens the argument.

E: it doesnot affect it .
  • Well, it might. If telephone equipment became more sophisticated between the 1970s and 1990, maybe the cost of phone calls should increase as well. On the other hand, "sophisticated" could imply higher efficiency, which would lead one to expect lower prices. It is unclear how this would affect the conclusion, and therefore answer choice (E) is incorrect.
There are two problems I can see from your approach to this problem: you misread what you read, likely because you are trying to read too fast; and also - you tend to forget what your job is. In solving a weaken question, the answers you found most attractive are the ones actually strengthening the argument. From your interpretation of the argument, it was clear you knew how to weaken it. But then somehow you started choosing answer choices that have the oppositie effect. Why is that?

Let me know if my explanations make sense. And thanks for showing us how you broke down the argument - this is incredibly helpful if we are to pinpoint precisely what you're doing wrong :-)
 Sdaoud17
  • Posts: 85
  • Joined: Apr 13, 2013
|
#9217
This is Great thank you :) :-D
 Harman
  • Posts: 17
  • Joined: Feb 17, 2015
|
#18396
After doing the question I read all of the explanations, the explanation for answer choice E however seemed odd. It says at first glance this answer choice is attractive because it implies that an increase in pay phone calls is justified. The authors conclusion is that pay phone companies should be allowed to double the price, and this is a weaken question. The explanation sounds like it is strengthening the authors conclusion, so why would it look attractive at first?

Then the explanation states that it is entirely possible that the technological advances have actually decreased the cost of the phone calls, this sounds like it would weaken the authors conclusion, but the explanation makes it seem like this answer choice is wrong because it does weaken it.

The price being lower weakens the authors conclusion, and the price being higher strengthens it; the explanation says the opposite.
I understand why this answer choice is incorrect, the explanation is confusing though.
It contradicts the explanation given for answer choice B.
 Steve Stein
PowerScore Staff
  • PowerScore Staff
  • Posts: 1153
  • Joined: Apr 11, 2011
|
#18404
Hey Harman,

Thanks for your message. The test makers have a few favorite wrong answers that they like to create, and one of them is the Opposite answer--when an answer choice has a clear effect on the argument--whether to strengthen or weaken--it can be an appealing answer choice whether right or wrong. With that said, I agree that the excerpt you referenced is somewhat confusing, but that will be clarified shortly.

Thanks again for your question and for your close reading! You're clearly committed to your preparation, which is vital if you are to maximize your score. Let me know whether this is clear, and if there are any other questions we can answer about your preparation--thanks!

~Steve
 Harman
  • Posts: 17
  • Joined: Feb 17, 2015
|
#18416
I read the Power Score bibles like a fundamentalist.
 mN2mmvf
  • Posts: 113
  • Joined: Jul 06, 2017
|
#38670
Why does the mere fact that the cost of ingredients for soda increased at a greater rate than telephone equipment weaken the case for raising the pay phone call price? Maybe that rate was 100% year over year for soda, but 95% for telephone equipment. The price increase will still be justified. Or, maybe the rate increased at only 0.002% for soda, and 0.001% for telephone equipment. In that case, the price increase would not be justified.
 Francis O'Rourke
PowerScore Staff
  • PowerScore Staff
  • Posts: 471
  • Joined: Mar 10, 2017
|
#38729
Answer choice (D) does not completely disprove the speaker's conclusion. As you said, it is possible that the difference in rate was marginal. It is also possible that the difference in rate was meaningful and would explain the increased costs in soda.

You should not assume that the correct Weaken answer will disprove the conclusion, or even severely weaken the conclusion. The correct answer may only provide the slightest piece of evidence for not accepting the author's conclusion. The more you look for severe weakening scenarios, the more answers like this one you will miss.

Get the most out of your LSAT Prep Plus subscription.

Analyze and track your performance with our Testing and Analytics Package.